---

问题

三维库仑势的形式为:

\[ V(\vec{r}) = \frac{1}{r} \]

按动量Fourier展开:

\[ V(\vec{r}) = \frac{1}{(2\pi)^3} \int_{-\infty}^{+\infty} \mathrm{d}^3\vec{k} \cdot V(\vec{k})\cdot e^{i\vec{k}\cdot\vec{r}} \]

求 \(V(\vec{k})\) 。

求解

\[ V{(\vec{k})} = \int_{-\infty}^{+\infty} \mathrm{d}^3\vec{r}\cdot V(\vec{r})\cdot e^{-i\vec{k}\cdot \vec{r}} = \int_0^{2\pi}\mathrm{d}\varphi \int_0^{+\infty}\mathrm{d}r\int_0^{\pi}\mathrm{d}\theta\cdot \frac{1}{r}e^{-ikr\cos\theta}r^2\sin\theta \\\\ =2\pi\int_0^{+\infty}\mathrm{d}r\int_0^{\pi}\mathrm{d}\theta\cdot e^{-ikr\cos\theta}r\sin\theta = -2\pi\int_0^{+\infty}\mathrm{d}r\int_1^{-1}\mathrm{d}\cos\theta\cdot e^{-ikr\cos\theta}r \\\\ =-2\pi\int_0^{+\infty}\mathrm{d}r\cdot \left[e^{-ikr\cos\theta}r\frac{1}{-ikr}\right]_{\cos\theta = 1}^{\cos\theta =-1} = 2\pi\int_0^{+\infty}\mathrm{d}r\cdot \left[(e^{ikr}-e^{-ikr})\frac{1}{ik}\right] \\\\ =\frac{2\pi}{ik}\left[\frac{e^{ikr}}{ik}-\frac{e^{-ikr}}{-ik}\right] _0^{+\infty} \]

但 \(e^{-ikr}\) 与 \(e^{-ikr}\) 在 \(+\infty\) 处发散。

使其收敛

为了解决发散问题,可在一开始令

\[ V(\vec{r}) = \frac{1}{r} = \lim_{\mu\rightarrow 0} \frac{e^{-\mu r}}{r} \]

所以

\[ V{(\vec{k})} = \lim_{\mu\rightarrow 0} 2\pi\int_0^{+\infty}\mathrm{d}r\cdot \left[(e^{(ik-\mu)r}-e^{(-ik-\mu)r})\frac{1}{ik}\right] \\\\ = \lim_{\mu\rightarrow 0}\frac{2\pi}{ik}\left[\frac{e^{(ik-\mu)r}}{ik-\mu r}-\frac{e^{(-ik-\mu)r}}{-ik-\mu r}\right] _0^{+\infty} \\\\ \] \[ = \lim_{\mu\rightarrow 0}\frac{2\pi}{ik}\left[\frac{-1}{ik-\mu }-\frac{-1}{-ik-\mu }\right] \\\\ =\lim_{\mu\rightarrow 0}\frac{2\pi}{ik}\frac{2ik}{k^2 + \mu ^2} =\lim_{\mu\rightarrow 0}\frac{4\pi}{k^2 + \mu ^2} \\\\ =\frac{4\pi}{k^2} \]

函数满足一致收敛,积分与极限才可交换顺序。

类似的变换

用同样的方法, 可以得到

\[ \lim_{\eta \to 0^+}\int \mathrm{d}^3\vec{r} . e^{\mathrm{i}\vec{k}\cdot\vec{r}} r e^{-r\eta} = \frac{8 \pi}{k^4} \] \[ \lim_{\eta \to 0^+}\int \mathrm{d}^3\vec{r} . e^{\mathrm{i}\vec{k}\cdot\vec{r}} e^{-r\eta} = 0 \]

用 Mathematica 可以验证.

Reference

  • 关于使其收敛的说明, 分别从物理的角度, 数学的角度阐述这样做的合理性和必要性: https://physics.stackexchange.com/questions/7462/fourier-transform-of-the-coulomb-potential